icone-challenge-math-OS

Solution pour le challenge 66


Il est connu que, pour tout x\in\left]-1,1\right] :

    \[\boxed{\ln\left(1+x\right)=\sum_{n=1}^{\infty}\frac{\left(-1\right)^{n-1}}{n}\thinspace x^{n}}\]

Si cette formule ne vous est pas familière, une preuve élémentaire en est donnée plus bas.

En prenant x=-\frac{1}{2}, on trouve :

    \[\ln\left(\frac{1}{2}\right)=\sum_{n=1}^{\infty}\frac{\left(-1\right)^{n-1}}{n}\thinspace\left(-\frac{1}{2}\right)^{n}\]

soit, après simplification :

    \[\boxed{\ln\left(2\right)=\sum_{n=1}^{\infty}\frac{1}{n\thinspace2^{n}}}\]

On peut exploiter ce développement en série pour obtenir des majorations non triviales de \ln\left(2\right). En effet, pour tout entier n\geqslant1 :

    \[\ln\left(2\right)=L_{n}+R_{n}\]

où l’on a posé :

    \[L_{n}=\sum_{k=1}^{n}\frac{1}{k\thinspace2^{k}}\qquad\text{et}\qquad R_{n}=\sum_{k=n+1}^{\infty}\frac{1}{k\thinspace2^{k}}\]

En outre :

    \[R_{n}<\frac{1}{n+1}\thinspace\sum_{k=n+1}^{\infty}\frac{1}{2^{k}}=\frac{1}{2^{n}\left(n+1\right)}\]

et par conséquent :

    \[\boxed{\forall n\in\mathbb{N}^{\star},\thinspace\ln\left(2\right)<L_{n}+\frac{1}{2^{n}\left(n+1\right)}}\]

➣ Pour n=1, cette majoration donne \ln\left(2\right)<\frac{3}{4}.

➣ Pour n=2, on obtient : \ln\left(2\right)<\frac{17}{24}.

➣ Pour n=3 : \ln\left(2\right)<\frac{67}{96}. Or \frac{67}{96}<\frac{7}{10} puisque 67\times10=670<672=7\times96. Nous avons donc bien prouvé que :

    \[\boxed{\ln\left(2\right)<\frac{7}{10}}\]

Ajoutons qu’avec ce qui précède, on peut aussi minorer \ln\left(2\right) par L_{n}, quel que soit n\geqslant1. Par exemple, pour n=6, il vient :

    \[\ln\left(2\right)>\frac{1}{2}+\frac{1}{8}+\frac{1}{24}+\frac{1}{64}+\frac{1}{160}+\frac{1}{384}>\frac{69}{100}\]

On pourra retenir l’encadrement à 10^{-2} près suivant :

    \[\boxed{0.69<\ln\left(2\right)<0.70}\]

Remarque

Dans la formule

    \[\ln\left(1+x\right)=\sum_{n=1}^{\infty}\frac{\left(-1\right)^{n-1}}{n}\thinspace x^{n}\]

qui est valable pour tout x\in\left]-1,1\right], on aurait pu tout simplement choisir x=1 pour obtenir :

    \[\ln(2)=\sum_{n=1}^\infty\frac{(-1)^{n-1}}{n}\]

puis en déduire une valeur approchée de \ln(2) en considérant une somme partielle suffisamment « longue ». Le problème est que cette série alternée converge trop lentement pour produire une approximation acceptable à peu de frais. Qu’on en juge (calculs réalisés avec Maple) :

    \[\ln(2)\simeq0,6931471805599453094172321\text{ à }10^{-25}\text{ près}\]

    \[\sum_{k=1}^{100}\frac{1}{k\,2^k}\simeq0,6931471805599453094172321\text{ à }10^{-25}\text{ près}\]

    \[\sum_{k=1}^{100}\frac{(-1)^{k-1}}{k}\simeq0,6881721793101952032446459\text{ à }10^{-25}\text{ près}\]

Avec 100 termes, la série alternée fournit un résultat médiocre (écart de l’ordre de 5\times10^{-3}) alors que l’autre série donne une approximation excellente (on peut vérifier que l’écart est inférieur à 10^{-32}).

Proposition

Pour tout x\in\left]-1,1\right] :

    \[\ln\left(1+x\right)=\sum_{n=1}^{\infty}\frac{\left(-1\right)^{n-1}}{n}\thinspace x^{n}\]

Preuve (cliquer pour déplier / replier)

On part de la formule explicite pour une somme géométrique.

Pour tout t\in\mathbb{R}-\left\{ -1\right\} et tout n\in\mathbb{N}^{\star} :

    \[\sum_{k=0}^{n-1}\left(-1\right)^{k}t^{k}=\frac{1-\left(-1\right)^{n}t^{n}}{1+t}\]

Ensuite, on intègre sur entre 0 et x, pour x>-1 :

    \[\sum_{k=0}^{n-1}\left(-1\right)^{k}\int_{0}^{x}t^{k}\thinspace dt=\int_{0}^{x}\frac{1}{1+t}\thinspace dt-\left(-1\right)^{n}\int_{0}^{x}\frac{t^{n}}{1+t}\thinspace dt\]

Autrement dit :

    \[\ln\left(1+x\right)=\sum_{k=0}^{n-1}\frac{\left(-1\right)^{k}}{k+1}\thinspace x^{k+1}+\left(-1^{n}\right)\int_{0}^{x}\frac{t^{n}}{1+t}\thinspace dt\]

Il reste à prouver que, si x\in\left]-1,1\right], alors cette dernière intégrale tend vers 0 lorsque n tend vers +\infty. Notons donc :

    \[R_{n}\left(x\right)=\int_{0}^{x}\frac{t^{n}}{1+t}\thinspace dt\]

et distinguons deux cas selon que x\in\left[0,1\right] ou que x\in\left]-1,0\right[.

1er cas – On suppose x\in\left[0,1\right], alors :

    \[0\leqslant R_{n}\left(x\right)\leqslant\int_{0}^{x}t^{n}\thinspace dt=\frac{x^{n+1}}{n+1}\leqslant\frac{1}{n+1}\]

et donc {\displaystyle \lim_{n\rightarrow\infty}R_{n}\left(x\right)=0.}

2ème cas – On suppose x\in\left]-1,0\right[ et l’on effectue le changement de variable s=-t :

    \[R_{n}\left(x\right)=-\int_{x}^{0}\frac{t^{n}}{1+t}\thinspace dt=\left(-1\right)^{n+1}\int_{0}^{-x}\frac{s^{n}}{1-s}\thinspace ds\]

donc :

    \[\left|R_{n}\left(x\right)\right|=\int_{0}^{-x}\frac{s^{n}}{1-s}\thinspace ds\leqslant\frac{1}{1+x}\int_{0}^{-x}\thinspace s^{n}\thinspace ds=\frac{\left(-x\right)^{n+1}}{\left(n+1\right)\left(1+x\right)}\leqslant\frac{1}{\left(n+1\right)\left(1+x\right)}\]

d’où, à nouveau, {\displaystyle \lim_{n\rightarrow\infty}R_{n}\left(x\right)=0.}


Pour consulter l’énoncé, c’est ici

Partager cet article

Laisser un commentaire